www.vorhilfe.de
Vorhilfe

Kostenlose Kommunikationsplattform für gegenseitige Hilfestellungen.
Hallo Gast!einloggen | registrieren ]
Startseite · Forum · Wissen · Kurse · Mitglieder · Team · Impressum
Forenbaum
^ Forenbaum
Status Englisch
  Status Grammatik
  Status Lektüre
  Status Korrekturlesen
  Status Übersetzung
  Status Sonstiges (Englisch)

Gezeigt werden alle Foren bis zur Tiefe 2

Navigation
 Startseite...
 Neuerdings beta neu
 Forum...
 vorwissen...
 vorkurse...
 Werkzeuge...
 Nachhilfevermittlung beta...
 Online-Spiele beta
 Suchen
 Verein...
 Impressum
Das Projekt
Server und Internetanbindung werden durch Spenden finanziert.
Organisiert wird das Projekt von unserem Koordinatorenteam.
Hunderte Mitglieder helfen ehrenamtlich in unseren moderierten Foren.
Anbieter der Seite ist der gemeinnützige Verein "Vorhilfe.de e.V.".
Partnerseiten
Weitere Fächer:

Open Source FunktionenplotterFunkyPlot: Kostenloser und quelloffener Funktionenplotter für Linux und andere Betriebssysteme
Forum "Zahlentheorie" - 3^m<2^k, aber ganz nah dran...
3^m<2^k, aber ganz nah dran... < Zahlentheorie < Algebra+Zahlentheo. < Hochschule < Mathe < Vorhilfe
Ansicht: [ geschachtelt ] | ^ Forum "Zahlentheorie"  | ^^ Alle Foren  | ^ Forenbaum  | Materialien

3^m<2^k, aber ganz nah dran...: Frage (beantwortet)
Status: (Frage) beantwortet Status 
Datum: 00:26 Fr 20.08.2010
Autor: reverend

Hallo allerseits,

diese Frage steht im Forum "Zahlentheorie", weil m.E. nur mit den Mitteln dieser Disziplin eine Antwort zu finden ist.

Das Problem ist eigentlich ziemlich einfach und tauchte neulich in einer fremden Anfrage auf, die auch gelöst werden konnte. Ich habe die Frage nur ein bisschen erweitert und auch einen Anwendungsgrund, der eine zusätzliche Verschärfung gebracht hat.

Aufgabe
Sei [mm] 3^m<2^k<3^{m+1} [/mm] mit [mm] k,m\in\IN. [/mm]
Wie "nahe" kann [mm] 3^m [/mm] an [mm] 2^k [/mm] herankommen? mod 8 ist leicht zu zeigen, dass [mm] 2^k-3^m\equiv 5\mod{8} [/mm] oder [mm] \equiv 7\mod{8} [/mm] sein muss. Sind aber 5 oder 7 (oder meinetwegen auch 13, 23 oder 55) möglich? Wenn ja: wie finde ich dann k und m? Wenn nein: wie zeige ich das?  


Ich habe mich versuchsweise und ganz unzahlentheoretisch über Logarithmen genähert und nach folgendem gesucht:

[mm] \bruch{a}{b}<\bruch{\ln{2}}{\ln{3}}<\bruch{a+s}{b} [/mm]

mit [mm] a,b\in{\IN} [/mm] und (hier verlassen wir die Gepflogenheiten der Zahlentheorie) [mm] s\in\IR, [/mm] sowie [mm] s<<\bruch{2^b}{10^n}, [/mm] wobei natürlich [mm] n\in\IN [/mm] gedacht ist. Allerdings suche ich ein Ergebnis, bei dem mindestens n>60, besser n>300 gilt.

Das ist mit einem Beispiel besser fassbar:
Es ist
[mm] 2^{301994}\approx 1,788588970*10^{90909} [/mm] und
[mm] 3^{190537}\approx 1,788588855*10^{90909} [/mm]

Das ist schon sehr nah, aber ich suche ein Ergebnis, das noch wesentlich näher ist. Kann ich es finden? Oder wenigstens seine Existenz nachweisen?

Wie nahe können Potenzen teilerfremder Zahlen einander kommen?

Übrigens: natürlich sind mir [mm] 2^2-3^1=1 [/mm] und [mm] 3^2-2^3=1 [/mm] bekannt, wie auch andere "Nähen", z.B. [mm] 13^3-3^7=10 [/mm] oder [mm] 7^6-3^{11}=502. [/mm] Das ist mir aber eben nicht genug...

Falls übrigens jemand Literatur zum Thema kennt oder eine Idee hat, wie ich sie finde, wäre mir sicher auch schon geholfen.

Danke für Euer Hirnschmalz, dies ist ja keine der üblichen Übungsaufgaben, aber ich hänge hier jetzt seit geraumer Zeit.
Hilfe wäre daher sehr willkommen. :-)

Herzliche Grüße
reverend

        
Bezug
3^m<2^k, aber ganz nah dran...: Antwort
Status: (Antwort) fertig Status 
Datum: 02:42 Fr 20.08.2010
Autor: felixf

Moin reverend,

> diese Frage steht im Forum "Zahlentheorie", weil m.E. nur
> mit den Mitteln dieser Disziplin eine Antwort zu finden
> ist.
>  
> Das Problem ist eigentlich ziemlich einfach und tauchte
> neulich in einer fremden Anfrage auf, die auch gelöst
> werden konnte. Ich habe die Frage nur ein bisschen
> erweitert und auch einen Anwendungsgrund, der eine
> zusätzliche Verschärfung gebracht hat.
>  
> Sei [mm]3^m<2^k<3^{m+1}[/mm] mit [mm]k,m\in\IN.[/mm]
>  Wie "nahe" kann [mm]3^m[/mm] an [mm]2^k[/mm] herankommen? mod 8 ist leicht
> zu zeigen, dass [mm]2^k-3^m\equiv 5\mod{8}[/mm] oder [mm]\equiv 7\mod{8}[/mm]Eingabefehler: "{" und "}" müssen immer paarweise auftreten, es wurde aber ein Teil ohne Entsprechung gefunden (siehe rote Markierung)


> sein muss. Sind aber 5 oder 7 (oder meinetwegen auch 13, 23
> oder 55) möglich? Wenn ja: wie finde ich dann k und m?
> Wenn nein: wie zeige ich das?

es ist $2^4 - 3^2 = 7$ und $2^5 - 3^3 = 5$. Weiterhin ist $2^8 - 3^5 = 13$. Es ist uebrigens auch 1 moeglich, mit $1 = 2^1 - 3^0$, und $-1$ mit $-1 = 2^2 - 3^1$.

Mir scheint es, als wenn die Differenz zwischen $2^k$ und $3^m$ immer groesser wird. Minimal ist sie ja fuer $m \in \{ m_1(k), m_2(k) \}$ mit $m_1(k) := \lfloor k \frac{\log 2}{\log 3} \rfloor$ und $m_2(k) := \lceil k \frac{\log 2}{\log 3} \rceil \}$.

Es sieht nun aus, als wenn $2^k - 3^{m_1(k)}$ sowie $3^{m_2(k)} - 2^k$ beide gegen unendlich gehen fuer $k \to \infty$. Spontan wuerde ich sagen (aufgrund von Tests bis $k = 1000$), dass die Differenz -- zumindest fuer die meisten $k$ -- nach unten durch $C \cdot 2^k / f(k)$ abgeschaetzt werden kann, wobei $f$ eine nicht ganz so stark wie $2^k$ wachsende Funktion ist ($f(k) = k$ scheint zu schwach wachsend zu sein).

Ich vermute, das ganze haengt stark mit der Frage zusammen, wie schnell $k \frac{\log 2}{\log 3}$ nahe an eine ganze Zahl herankommen kann. Oder anders gesagt, wie nahe fuer begrenztes $k$ $\frac{\log 2}{\log 3}$ an einen Bruch mit Nenner $k$ herankommen kann.

Da $\frac{\log 2}{\log 3}$ transzendent sein duerfte, muesste man hier vermutlich nach Resultaten aus der Theorie der Approximation transzendenter Zahlen ausschau halten. Davon hab ich leider nicht so viel Ahnung...


>
> Ich habe mich versuchsweise und ganz unzahlentheoretisch
> über Logarithmen genähert und nach folgendem gesucht:

So unzahlentheoretisch ist es auch wieder nicht ;-)

> [mm]\bruch{a}{b}<\bruch{\ln{2}}{\ln{3}}<\bruch{a+s}{b}[/mm]
>  
> mit [mm]a,b\in{\IN}[/mm] und (hier verlassen wir die Gepflogenheiten
> der Zahlentheorie) [mm]s\in\IR,[/mm] sowie [mm]s<<\bruch{2^b}{10^n},[/mm]
> wobei natürlich [mm]n\in\IN[/mm] gedacht ist. Allerdings suche ich
> ein Ergebnis, bei dem mindestens n>60, besser n>300 gilt.

Ich glaub nicht, dass wir hier die Gepflogenheiten der Zahlentheorie hinter uns lassen. Hoechstens die der elementaren Zahlentheorie :)

> Das ist mit einem Beispiel besser fassbar:
>  Es ist
> [mm]2^{301994}\approx 1,788588970*10^{90909}[/mm] und
>  [mm]3^{190537}\approx 1,788588855*10^{90909}[/mm]

Hier gilt uebrigens [mm] $\frac{2^{301994} - 3^{190537}}{2^{301994}} \cdot [/mm] 301994 [mm] \approx [/mm] 0.01948087817$. Das Minimum von [mm] $\frac{\min\{ 2^k - 3^{m_1(k)}, 3^{m_2(k)} - 2^k \}}{2^k} \cdot [/mm] k$ liegt fuer $k = 1, [mm] \dots, [/mm] 10000$ bei [mm] $\approx [/mm] 0.046012436869$.

(Wen's interessiert: Digits := 1000; min(seq(evalf((2^n - 3^floor(n * log(2) / log(3))) / 2^n * n), n = 1 .. 10000), seq(evalf(abs(2^n - 3^ceil(n * log(2) / log(3))) / 2^n * n), n = 1 .. 10000)); in MAPLE.)

Also ist $f(t) = t$ vielleicht doch gar nicht so schlecht.

> Das ist schon sehr nah, aber ich suche ein Ergebnis, das
> noch wesentlich näher ist. Kann ich es finden? Oder
> wenigstens seine Existenz nachweisen?

Oder: kann man zeigen, dass es nicht viel besser geht?

Mehr kann ich dazu erstmal nicht sagen...

LG Felix


Bezug
                
Bezug
3^m<2^k, aber ganz nah dran...: Mitteilung
Status: (Mitteilung) Reaktion unnötig Status 
Datum: 14:54 Fr 20.08.2010
Autor: reverend

Moin Felix,

danke schonmal. Ich komme aber erst viel später zu einer detaillierteren Reaktion, vielleicht sogar erst morgen. Bin gerade auf Achse.

Ob die "schlichte" rationale Approximation transzendenter Zahlen hier genügt, weiß ich noch nicht. Ich denk mal ein bisschen drüber nach, während mein Seminar Arbeitsphasen hat...

Liebe Grüße
reverend

Bezug
                        
Bezug
3^m<2^k, aber ganz nah dran...: Mitteilung
Status: (Mitteilung) Reaktion unnötig Status 
Datum: 16:54 Fr 20.08.2010
Autor: felixf

Moin reverend,

> Ob die "schlichte" rationale Approximation transzendenter
> Zahlen hier genügt, weiß ich noch nicht. Ich denk mal ein
> bisschen drüber nach, während mein Seminar Arbeitsphasen
> hat...

eine "schlichte" Approximation tut es auch nicht so gut. Es muss schon eine "sehr gute" Approximation sein. Je besser die Approximation, desto kleiner der Abstand zwischen [mm] $2^k$ [/mm] und [mm] $3^m$. [/mm]

LG Felix


Bezug
                                
Bezug
3^m<2^k, aber ganz nah dran...: Mitteilung
Status: (Mitteilung) Reaktion unnötig Status 
Datum: 02:00 Di 24.08.2010
Autor: reverend

Hallo Felix,

ich bin dann doch unerwartet vom Arbeitsanfall überrollt worden. Sorry.

> eine "schlichte" Approximation tut es auch nicht so gut. Es
> muss schon eine "sehr gute" Approximation sein. Je besser
> die Approximation, desto kleiner der Abstand zwischen [mm]2^k[/mm]
> und [mm]3^m[/mm].

Ja, schon klar. Dennoch hoffe ich noch auf einen Weg, der mit einbezieht, dass die Approximation auf der einen Seite seeehr viel besser sein muss als auf der anderen.

Im Moment suche ich mit einem anderen Ansatz nach Teilbarkeiten - aber noch bin ich die gezielte Annäherung nicht los. Vielleicht findet sich da doch etwas.

Danke jedenfalls für Deine ermutigende Reaktion!

lg, reverend

Bezug
        
Bezug
3^m<2^k, aber ganz nah dran...: Antwort
Status: (Antwort) fertig Status 
Datum: 16:08 Fr 20.08.2010
Autor: abakus


> Hallo allerseits,
>  
> diese Frage steht im Forum "Zahlentheorie", weil m.E. nur
> mit den Mitteln dieser Disziplin eine Antwort zu finden
> ist.
>  
> Das Problem ist eigentlich ziemlich einfach und tauchte
> neulich in einer fremden Anfrage auf, die auch gelöst
> werden konnte. Ich habe die Frage nur ein bisschen
> erweitert und auch einen Anwendungsgrund, der eine
> zusätzliche Verschärfung gebracht hat.
>  
> Sei [mm]3^m<2^k<3^{m+1}[/mm] mit [mm]k,m\in\IN.[/mm]
>  Wie "nahe" kann [mm]3^m[/mm] an [mm]2^k[/mm] herankommen?

Hallo,
für gerade k mit k>2 auf alle Fälle nicht so nahe, dass  [mm] 3^m+1=2^k [/mm] gilt.
Denn dann würde auch [mm] 3^m=2^k-1 =(2^\bruch{k}{2}-1)(2^\bruch{k}{2}+1)gelten. [/mm] Eine Potenz von 3 kann aber nicht in ein Produkt zerlegt werden, dessen Faktoren nur den Abstand 2 haben (Ausnahme: 1*3).
Gruß Abakus

Bezug
                
Bezug
3^m<2^k, aber ganz nah dran...: Mitteilung
Status: (Mitteilung) Reaktion unnötig Status 
Datum: 02:10 Di 24.08.2010
Autor: reverend

Hallo abakus,

danke, dass Du Dich mit meiner Anfrage auseinandergesetzt hast! Deine Einlassung ist natürlich vollkommen richtig, hilft mir aber gerade nicht so sehr weiter. Dieses Ergebnis erzielt man genauso leicht mit einer Betrachtung mod 8. Dennoch erinnerst Du mich daran, dass viele Wege nach Rom führen - jedenfalls an dieser Stelle noch. Für sehr hohe Exponenten mag es aber auch noch andere Betrachtungsweisen geben, die ich bisher vernachlässigt habe.

Vielleicht habe ich aber auch einfach meine Frage nicht präzise genug gestellt. Das Problem, an dem ich gerade arbeite, will ich gar nicht als ganzes darstellen, sondern nur meine offenen Fragen formulieren. Womöglich liegt hier aber das Problem hinsichtlich der Ermöglichung von Hilfestellung. Bei anderen Anfragen gehöre ich ja auch zu denen, die um die möglichst vollständige ursprüngliche Aufgabenstellung bitten. Im aktuellen Fall bin ich aber noch nicht so weit, die selbstgestellte Aufgabe wirklich sauber formulieren zu können.

Danke also vorerst für Deine Reaktion. Wenn es Neues in dieser Hinsicht gibt, wirst Du sicher schnell davon wissen.

Liebe Grüße
reverend

PS: Ab 12.9. bin ich ein paar Tage in Thüringen, also nicht weit weg, und bei mir zeitlich ziemlich beweglich... Vielleicht sehen wir uns aber auch beim Mod-Treffen 2011? Das ist wohl mehr ein impliziter Vorschlag als eine Frage...

Bezug
Ansicht: [ geschachtelt ] | ^ Forum "Zahlentheorie"  | ^^ Alle Foren  | ^ Forenbaum  | Materialien


^ Seitenanfang ^
www.englischraum.de
[ Startseite | Forum | Wissen | Kurse | Mitglieder | Team | Impressum ]